11.07.2015 Views

June-2007-LSAT-PrepTest

June-2007-LSAT-PrepTest

June-2007-LSAT-PrepTest

SHOW MORE
SHOW LESS

Create successful ePaper yourself

Turn your PDF publications into a flip-book with our unique Google optimized e-Paper software.

www.cambridgelsat.comA Publication of the Law School Admission Council,The Producers of the <strong>LSAT</strong> ®THE OFFICIAL <strong>LSAT</strong>PREPTEST®• Form 8LSN75JUNE <strong>2007</strong>LSAC.org


1 -2- 2www.cambridgelsat.com1 1SECTION ITime—35 minutes23 QuestionsDirections: Each group of questions in this section is based on a set of conditions. In answering some of the questions, it may beuseful to draw a rough diagram. Choose the response that most accurately and completely answers each question and blackenthe corresponding space on your answer sheet.Questions 1–5A company employee generates a series of five-digit productcodes in accordance with the following rules:The codes use the digits 0, 1, 2, 3, and 4, and no others.Each digit occurs exactly once in any code.The second digit has a value exactly twice that of thefirst digit.The value of the third digit is less than the value of thefifth digit.1. If the last digit of an acceptable product code is 1, itmust be true that the(A) first digit is 2(B) second digit is 0(C) third digit is 3(D) fourth digit is 4(E) fourth digit is 02. Which one of the following must be true about anyacceptable product code?(A)(B)(C)(D)(E)The digit 1 appears in some position before thedigit 2.The digit 1 appears in some position before thedigit 3.The digit 2 appears in some position before thedigit 3.The digit 3 appears in some position before thedigit 0.The digit 4 appears in some position before thedigit 3.4. Any of the following pairs could be the third andfourth digits, respectively, of an acceptable productcode, EXCEPT:(A) 0, 1(B) 0, 3(C) 1, 0(D) 3, 0(E) 3, 45. Which one of the following must be true about anyacceptable product code?(A) There is exactly one digit between the digit 0and the digit 1.(B) There is exactly one digit between the digit 1and the digit 2.(C) There are at most two digits between the digit 1and the digit 3.(D) There are at most two digits between the digit 2and the digit 3.(E) There are at most two digits between the digit 2and the digit 4.GO ON TO THE NEXT PAGE.3. If the third digit of an acceptable product code is not 0,which one of the following must be true?(A) The second digit of the product code is 2.(B) The third digit of the product code is 3.(C) The fourth digit of the product code is 0.(D) The fifth digit of the product code is 3.(E) The fifth digit of the product code is 1.


Questions 6–10Exactly three films—Greed, Harvest, and Limelight—areshown during a film club’s festival held on Thursday, Friday,and Saturday. Each film is shown at least once during thefestival but never more than once on a given day. On each dayat least one film is shown. Films are shown one at a time. Thefollowing conditions apply:On Thursday Harvest is shown, and no film is shown afterit on that day.On Friday either Greed or Limelight, but not both, isshown, and no film is shown after it on that day.On Saturday either Greed or Harvest, but not both, isshown, and no film is shown after it on that day.6. Which one of the following could be a complete andaccurate description of the order in which the films areshown at the festival?(A)(B)(C)(D)(E)Thursday: Limelight, then Harvest; Friday:Limelight; Saturday: HarvestThursday: Harvest; Friday: Greed, thenLimelight; Saturday: Limelight, then GreedThursday: Harvest; Friday: Limelight; Saturday:Limelight, then GreedThursday: Greed, then Harvest, then Limelight;Friday: Limelight; Saturday: GreedThursday: Greed, then Harvest; Friday:Limelight, then Harvest; Saturday: Harvest7. Which one of the following CANNOT be true?(A)(B)(C)(D)(E)Harvest is the last film shown on each day of thefestival.Limelight is shown on each day of the festival.Greed is shown second on each day of thefestival.A different film is shown first on each day of thefestival.A different film is shown last on each day of thefestival.www.cambridgelsat.com1 1-3-18. If Limelight is never shown again during the festivalonce Greed is shown, then which one of the following isthe maximum number of film showings that could occurduring the festival?(A)(B)(C)(D)(E)threefourfivesixseven9. If Greed is shown exactly three times, Harvest is shownexactly twice, and Limelight is shown exactly once, thenwhich one of the following must be true?(A)(B)(C)(D)(E)All three films are shown on Thursday.Exactly two films are shown on Saturday.Limelight and Harvest are both shown onThursday.Greed is the only film shown on Saturday.Harvest and Greed are both shown on Friday.10. If Limelight is shown exactly three times, Harvest isshown exactly twice, and Greed is shown exactly once,then which one of the following is a complete andaccurate list of the films that could be the first filmshown on Thursday?(A)(B)(C)(D)(E)HarvestLimelightGreed, HarvestGreed, LimelightGreed, Harvest, LimelightGO ON TO THE NEXT PAGE.3


2 -6- 6www.cambridgelsat.com2 2 2SECTION IITime—35 minutes25 QuestionsDirections: The questions in this section are based on the reasoning contained in brief statements or passages. For somequestions, more than one of the choices could conceivably answer the question. However, you are to choose the best answer; thatis, the response that most accurately and completely answers the question. You should not make assumptions that are bycommonsense standards implausible, superfluous, or incompatible with the passage. After you have chosen the best answer,blacken the corresponding space on your answer sheet.1. Economist: Every business strives to increase itsproductivity, for this increases profits for theowners and the likelihood that the business willsurvive. But not all efforts to increaseproductivity are beneficial to the business as awhole. Often, attempts to increase productivitydecrease the number of employees, which clearlyharms the dismissed employees as well as thesense of security of the retained employees.Which one of the following most accurately expressesthe main conclusion of the economist’s argument?(A)(B)(C)(D)(E)If an action taken to secure the survival of abusiness fails to enhance the welfare of thebusiness’s employees, that action cannot begood for the business as a whole.Some measures taken by a business to increaseproductivity fail to be beneficial to the businessas a whole.Only if the employees of a business are also itsowners will the interests of the employees andowners coincide, enabling measures that willbe beneficial to the business as a whole.There is no business that does not make effortsto increase its productivity.Decreasing the number of employees in abusiness undermines the sense of security ofretained employees.2. All Labrador retrievers bark a great deal. All SaintBernards bark infrequently. Each of Rosa’s dogs is across between a Labrador retriever and a Saint Bernard.Therefore, Rosa’s dogs are moderate barkers.Which one of the following uses flawed reasoning thatmost closely resembles the flawed reasoning used inthe argument above?(A)(B)(C)(D)(E)All students who study diligently make goodgrades. But some students who do not studydiligently also make good grades. Jane studiessomewhat diligently. Therefore, Jane makessomewhat good grades.All type A chemicals are extremely toxic tohuman beings. All type B chemicals arenontoxic to human beings. This householdcleaner is a mixture of a type A chemical anda type B chemical. Therefore, this householdcleaner is moderately toxic.All students at Hanson School live in GreenCounty. All students at Edwards School live inWinn County. Members of the Perry familyattend both Hanson and Edwards. Therefore,some members of the Perry family live inGreen County and some live in Winn County.All transcriptionists know shorthand. Allengineers know calculus. Bob has worked bothas a transcriptionist and as an engineer.Therefore, Bob knows both shorthand andcalculus.All of Kenisha’s dresses are very well made.All of Connie’s dresses are very badly made.Half of the dresses in this closet are very wellmade, and half of them are very badly made.Therefore, half of the dresses in this closet areKenisha’s and half of them are Connie’s.GO ON TO THE NEXT PAGE.


3. A century in certain ways is like a life, and as the endof a century approaches, people behave toward thatcentury much as someone who is nearing the end oflife does toward that life. So just as people in their lastyears spend much time looking back on the events oftheir life, people at a century’s end _______.Which one of the following most logically completesthe argument?(A)(B)(C)(D)(E)reminisce about their own livesfear that their own lives are about to endfocus on what the next century will bringbecome very interested in the history of thecentury just endingreflect on how certain unfortunate events of thecentury could have been avoided4. Consumer: The latest Connorly Report suggests thatOcksenfrey prepackaged meals are virtuallydevoid of nutritional value. But the ConnorlyReport is commissioned by Danto Foods,Ocksenfrey’s largest corporate rival, and earlydrafts of the report are submitted for approval toDanto Foods’ public relations department.Because of the obvious bias of this report, it isclear that Ocksenfrey’s prepackaged meals reallyare nutritious.The reasoning in the consumer’s argument is mostvulnerable to criticism on the grounds that theargument(A)(B)(C)(D)(E)treats evidence that there is an apparent bias asevidence that the Connorly Report’s claims arefalsedraws a conclusion based solely on anunrepresentative sample of Ocksenfrey’sproductsfails to take into account the possibility thatOcksenfrey has just as much motivation tocreate negative publicity for Danto as Dantohas to create negative publicity for Ocksenfreyfails to provide evidence that Danto Foods’prepackaged meals are not more nutritious thanOcksenfrey’s arepresumes, without providing justification, thatDanto Foods’ public relations departmentwould not approve a draft of a report that washostile to Danto Foods’ productswww.cambridgelsat.com5. Scientist: Earth’s average annual temperature hasincreased by about 0.5 degrees Celsius over thelast century. This warming is primarily the resultof the buildup of minor gases in the atmosphere,blocking the outward flow of heat from theplanet.2 2 2-7-2Which one of the following, if true, would count asevidence against the scientist’s explanation of Earth’swarming?(A) Only some of the minor gases whose presencein the atmosphere allegedly resulted in thephenomenon described by the scientist wereproduced by industrial pollution.(B) Most of the warming occurred before 1940,while most of the buildup of minor gases inthe atmosphere occurred after 1940.(C) Over the last century, Earth received slightlymore solar radiation in certain years than it didin others.(D) Volcanic dust and other particles in theatmosphere reflect much of the Sun’s radiationback into space before it can reach Earth’ssurface.(E) The accumulation of minor gases in theatmosphere has been greater over the lastcentury than at any other time in Earth’shistory.6. An undergraduate degree is necessary for appointmentto the executive board. Further, no one with a felonyconviction can be appointed to the board. Thus,Murray, an accountant with both a bachelor’s and amaster’s degree, cannot be accepted for the position ofExecutive Administrator, since he has a felonyconviction.The argument’s conclusion follows logically if whichone of the following is assumed?(A)(B)(C)(D)(E)Anyone with a master’s degree and without afelony conviction is eligible for appointment tothe executive board.Only candidates eligible for appointment to theexecutive board can be accepted for theposition of Executive Administrator.An undergraduate degree is not necessary foracceptance for the position of ExecutiveAdministrator.If Murray did not have a felony conviction, hewould be accepted for the position ofExecutive Administrator.The felony charge on which Murray wasconvicted is relevant to the duties of theposition of Executive Administrator.7GO ON TO THE NEXT PAGE.


2 -8- 87. Ethicist: The most advanced kind of moral motivationis based solely on abstract principles. This formof motivation is in contrast with calculated selfinterestor the desire to adhere to societal normsand conventions.The actions of which one of the following individualsexhibit the most advanced kind of moral motivation, asdescribed by the ethicist?(A)(B)(C)(D)(E)Bobby contributed money to a local charityduring a charity drive at work because heworried that not doing so would make himlook stingy.Wes contributed money to a local charity duringa charity drive at work because he believedthat doing so would improve his employer’sopinion of him.Donna’s employers engaged in an illegal butprofitable practice that caused serious damageto the environment. Donna did not report thispractice to the authorities, out of fear that heremployers would retaliate against her.Jadine’s employers engaged in an illegal butprofitable practice that caused serious damageto the environment. Jadine reported thispractice to the authorities out of a belief thatprotecting the environment is always moreimportant than monetary profit.Leigh’s employers engaged in an illegal butprofitable practice that caused serious damageto the environment. Leigh reported this practiceto the authorities only because severalcolleagues had been pressuring her to do so.8. Proponents of the electric car maintain that when thetechnical problems associated with its battery designare solved, such cars will be widely used and, becausethey are emission-free, will result in an abatement ofthe environmental degradation caused by autoemissions. But unless we dam more rivers, theelectricity to charge these batteries will come fromnuclear or coal-fired power plants. Each of these threepower sources produces considerable environmentaldamage. Thus, the electric car _______.Which one of the following most logically completesthe argument?(A)(B)(C)(D)(E)will have worse environmental consequencesthan its proponents may believewill probably remain less popular than othertypes of carsrequires that purely technical problems besolved before it can succeedwill increase the total level of emissions ratherthan reduce itwill not produce a net reduction inenvironmental degradationwww.cambridgelsat.com2 2 29. Although video game sales have increased steadily overthe past 3 years, we can expect a reversal of this trendin the very near future. Historically, over three quartersof video games sold have been purchased by peoplefrom 13 to 16 years of age, and the number of peoplein this age group is expected to decline steadily overthe next 10 years.Which one of the following, if true, would mostseriously weaken the argument?(A)(B)(C)(D)(E)Most people 17 years old or older have neverpurchased a video game.Video game rentals have declined over the past3 years.New technology will undoubtedly make entirelynew entertainment options available over thenext 10 years.The number of different types of video gamesavailable is unlikely to decrease in the nearfuture.Most of the people who have purchased videogames over the past 3 years are over the ageof 16.10. Double-blind techniques should be used wheneverpossible in scientific experiments. They help preventthe misinterpretations that often arise due toexpectations and opinions that scientists already hold,and clearly scientists should be extremely diligent intrying to avoid such misinterpretations.Which one of the following most accurately expressesthe main conclusion of the argument?(A)(B)(C)(D)(E)Scientists’ objectivity may be impeded byinterpreting experimental evidence on the basisof expectations and opinions that they alreadyhold.It is advisable for scientists to use double-blindtechniques in as high a proportion of theirexperiments as they can.Scientists sometimes neglect to adequatelyconsider the risk of misinterpreting evidence onthe basis of prior expectations and opinions.Whenever possible, scientists should refrainfrom interpreting evidence on the basis ofpreviously formed expectations andconvictions.Double-blind experimental techniques are oftenan effective way of ensuring scientificobjectivity.GO ON TO THE NEXT PAGE.


11. It is now a common complaint that the electronicmedia have corroded the intellectual skills required andfostered by the literary media. But several centuriesago the complaint was that certain intellectual skills,such as the powerful memory and extemporaneouseloquence that were intrinsic to oral culture, were beingdestroyed by the spread of literacy. So, what awaits usis probably a mere alteration of the human mind ratherthan its devolution.The reference to the complaint of several centuries agothat powerful memory and extemporaneous eloquencewere being destroyed plays which one of the followingroles in the argument?(A)(B)(C)(D)(E)evidence supporting the claim that theintellectual skills fostered by the literary mediaare being destroyed by the electronic mediaan illustration of the general hypothesis beingadvanced that intellectual abilities areinseparable from the means by which peoplecommunicatean example of a cultural change that did notnecessarily have a detrimental effect on thehuman mind overallevidence that the claim that the intellectualskills required and fostered by the literarymedia are being lost is unwarrantedpossible evidence, mentioned and thendismissed, that might be cited by supporters ofthe hypothesis being criticizedwww.cambridgelsat.com12. Suppose I have promised to keep a confidence andsomeone asks me a question that I cannot answertruthfully without thereby breaking the promise.Obviously, I cannot both keep and break the samepromise. Therefore, one cannot be obliged both toanswer all questions truthfully and to keep allpromises.2 2 2-9-2Which one of the following arguments is most similarin its reasoning to the argument above?(A)(B)(C)(D)(E)It is claimed that we have the unencumberedright to say whatever we want. It is alsoclaimed that we have the obligation to be civilto others. But civility requires that we notalways say what we want. So, it cannot be trueboth that we have the unencumbered right tosay whatever we want and that we have theduty to be civil.Some politicians could attain popularity withvoters only by making extravagant promises;this, however, would deceive the people. So,since the only way for some politicians to bepopular is to deceive, and any politician needsto be popular, it follows that some politiciansmust deceive.If we put a lot of effort into making this reportlook good, the client might think we did sobecause we believed our proposal would notstand on its own merits. On the other hand, ifwe do not try to make the report look good,the client might think we are not serious abouther business. So, whatever we do, we risk hercriticism.If creditors have legitimate claims against abusiness and the business has the resources topay those debts, then the business is obliged topay them. Also, if a business has obligations topay debts, then a court will force it to paythem. But the courts did not force this businessto pay its debts, so either the creditors did nothave legitimate claims or the business did nothave sufficient resources.If we extend our business hours, we will eitherhave to hire new employees or have existingemployees work overtime. But both newemployees and additional overtime woulddramatically increase our labor costs. Wecannot afford to increase labor costs, so wewill have to keep our business hours as theystand.9GO ON TO THE NEXT PAGE.


2 -10- 1013. Standard aluminum soft-drink cans do not vary in theamount of aluminum that they contain. Fifty percent ofthe aluminum contained in a certain group (M) ofstandard aluminum soft-drink cans was recycled fromanother group (L) of used, standard aluminum softdrinkcans. Since all the cans in L were recycled intocans in M and since the amount of material other thanaluminum in an aluminum can is negligible, it followsthat M contains twice as many cans as L.The conclusion of the argument follows logically ifwhich one of the following is assumed?(A)(B)(C)(D)(E)The aluminum in the cans of M cannot berecycled further.Recycled aluminum is of poorer quality thanunrecycled aluminum.All of the aluminum in an aluminum can isrecovered when the can is recycled.None of the soft-drink cans in group L had beenmade from recycled aluminum.Aluminum soft-drink cans are more easilyrecycled than are soft-drink cans made fromother materials.14. A cup of raw milk, after being heated in a microwaveoven to 50 degrees Celsius, contains half its initialconcentration of a particular enzyme, lysozyme. If,however, the milk reaches that temperature throughexposure to a conventional heat source of 50 degreesCelsius, it will contain nearly all of its initialconcentration of the enzyme. Therefore, what destroysthe enzyme is not heat but microwaves, which generateheat.Which one of the following, if true, most seriouslyweakens the argument?(A)(B)(C)(D)(E)Heating raw milk in a microwave oven to atemperature of 100 degrees Celsius destroysnearly all of the lysozyme initially present inthat milk.Enzymes in raw milk that are destroyed throughexcessive heating can be replaced by addingenzymes that have been extracted from othersources.A liquid exposed to a conventional heat sourceof exactly 50 degrees Celsius will reach thattemperature more slowly than it would if itwere exposed to a conventional heat sourcehotter than 50 degrees Celsius.Milk that has been heated in a microwave ovendoes not taste noticeably different from milkthat has been briefly heated by exposure to aconventional heat source.Heating any liquid by microwave creates smallzones within it that are much hotter than theoverall temperature that the liquid willultimately reach.www.cambridgelsat.com2 2 215. A new government policy has been developed to avoidmany serious cases of influenza. This goal will beaccomplished by the annual vaccination of high-riskindividuals: everyone 65 and older as well as anyonewith a chronic disease that might cause them toexperience complications from the influenza virus.Each year’s vaccination will protect only against thestrain of the influenza virus deemed most likely to beprevalent that year, so every year it will be necessaryfor all high-risk individuals to receive a vaccine for adifferent strain of the virus.Which one of the following is an assumption thatwould allow the conclusion above to be properly drawn?(A)(B)(C)(D)(E)The number of individuals in the high-riskgroup for influenza will not significantlychange from year to year.The likelihood that a serious influenza epidemicwill occur varies from year to year.No vaccine for the influenza virus protectsagainst more than one strain of that virus.Each year the strain of influenza virus deemedmost likely to be prevalent will be one that hadnot previously been deemed most likely to beprevalent.Each year’s vaccine will have fewer side effectsthan the vaccine of the previous year since thetechnology for making vaccines will constantlyimprove.GO ON TO THE NEXT PAGE.


16. Taylor: Researchers at a local university claim that61 percent of the information transferred during aconversation is communicated through nonverbalsignals. But this claim, like all suchmathematically precise claims, is suspect, becauseclaims of such exactitude could never beestablished by science.Sandra: While precision is unobtainable in many areasof life, it is commonplace in others. Manyscientific disciplines obtain extremely preciseresults, which should not be doubted merelybecause of their precision.The statements above provide the most support forholding that Sandra would disagree with Taylor aboutwhich one of the following statements?(A)(B)(C)(D)(E)Research might reveal that 61 percent of theinformation taken in during a conversation iscommunicated through nonverbal signals.It is possible to determine whether 61 percent ofthe information taken in during a conversationis communicated through nonverbal signals.The study of verbal and nonverbalcommunication is an area where one cannotexpect great precision in one’s research results.Some sciences can yield mathematically preciseresults that are not inherently suspect.If inherently suspect claims are usually false,then the majority of claims made by scientistsare false as well.17. Hospital executive: At a recent conference on nonprofitmanagement, several computer expertsmaintained that the most significant threat facedby large institutions such as universities andhospitals is unauthorized access to confidentialdata. In light of this testimony, we should makethe protection of our clients’ confidentiality ourhighest priority.The hospital executive’s argument is most vulnerable towhich one of the following objections?(A)(B)(C)(D)(E)The argument confuses the causes of a problemwith the appropriate solutions to that problem.The argument relies on the testimony of expertswhose expertise is not shown to be sufficientlybroad to support their general claim.The argument assumes that a correlationbetween two phenomena is evidence that one isthe cause of the other.The argument draws a general conclusion abouta group based on data about anunrepresentative sample of that group.The argument infers that a property belonging tolarge institutions belongs to all institutions.www.cambridgelsat.com18. Modern science is built on the process of posinghypotheses and testing them against observations—inessence, attempting to show that the hypotheses areincorrect. Nothing brings more recognition thanoverthrowing conventional wisdom. It is accordinglyunsurprising that some scientists are skeptical of thewidely accepted predictions of global warming. What isinstead remarkable is that with hundreds of researchersstriving to make breakthroughs in climatology, veryfew find evidence that global warming is unlikely.2 2 2-11-2The information above provides the most support forwhich one of the following statements?(A)(B)(C)(D)(E)Most scientists who are reluctant to accept theglobal warming hypothesis are not acting inaccordance with the accepted standards ofscientific debate.Most researchers in climatology have substantialmotive to find evidence that would discreditthe global warming hypothesis.There is evidence that conclusively shows thatthe global warming hypothesis is true.Scientists who are skeptical about globalwarming have not offered any alternativehypotheses to explain climatological data.Research in global warming is primarily drivenby a desire for recognition in the scientificcommunity.19. Historian: The Land Party achieved its only nationalvictory in Banestria in 1935. It received most ofits support that year in rural and semirural areas,where the bulk of Banestria’s population lived atthe time. The economic woes of the yearssurrounding that election hit agricultural andsmall business interests the hardest, and the LandParty specifically targeted those groups in 1935. Iconclude that the success of the Land Party thatyear was due to the combination of the LandParty’s specifically addressing the concerns ofthese groups and the depth of the economicproblems people in these groups were facing.Each of the following, if true, strengthens thehistorian’s argument EXCEPT:(A)(B)(C)(D)(E)In preceding elections the Land Party made noattempt to address the interests of economicallydistressed urban groups.Voters are more likely to vote for a politicalparty that focuses on their problems.The Land Party had most of its successes when therewas economic distress in the agricultural sector.No other major party in Banestria specificallyaddressed the issues of people who lived insemirural areas in 1935.The greater the degree of economic distresssomeone is in, the more likely that person isto vote.GO ON TO THE NEXT PAGE.11


2 -12- 1220. Gamba: Muñoz claims that the Southwest HopevilleNeighbors Association overwhelmingly opposesthe new water system, citing this as evidence ofcitywide opposition. The association did pass aresolution opposing the new water system, butonly 25 of 350 members voted, with 10 in favorof the system. Furthermore, the 15 opposingvotes represent far less than 1 percent ofHopeville’s population. One should not assumethat so few votes represent the view of themajority of Hopeville’s residents.Of the following, which one most accurately describesGamba’s strategy of argumentation?(A)(B)(C)(D)(E)questioning a conclusion based on the results ofa vote, on the grounds that people with certainviews are more likely to votequestioning a claim supported by statistical databy arguing that statistical data can bemanipulated to support whatever view theinterpreter wants to supportattempting to refute an argument by showingthat, contrary to what has been claimed, thetruth of the premises does not guarantee thetruth of the conclusioncriticizing a view on the grounds that the viewis based on evidence that is in principleimpossible to disconfirmattempting to cast doubt on a conclusion byclaiming that the statistical sample on whichthe conclusion is based is too small to bedependablewww.cambridgelsat.com2 2 222. Editorialist: News media rarely cover local politicsthoroughly, and local political business is usuallyconducted secretively. These factors each tend toisolate local politicians from their electorates.This has the effect of reducing the chance thatany particular act of resident participation willelicit a positive official response, which in turndiscourages resident participation in local politics.Which one of the following is most strongly supportedby the editorialist’s statements?(A)(B)(C)(D)(E)Particular acts of resident participation would belikely to elicit a positive response from localpoliticians if those politicians were less isolatedfrom their electorate.Local political business should be conductedless secretively because this would avoiddiscouraging resident participation in localpolitics.The most important factor influencing aresident’s decision as to whether to participatein local politics is the chance that theparticipation will elicit a positive officialresponse.More-frequent thorough coverage of localpolitics would reduce at least one source ofdiscouragement from resident participation inlocal politics.If resident participation in local politics werenot discouraged, this would cause localpoliticians to be less isolated from theirelectorate.21. Driver: My friends say I will one day have an accidentbecause I drive my sports car recklessly. But Ihave done some research, and apparentlyminivans and larger sedans have very lowaccident rates compared to sports cars. So tradingmy sports car in for a minivan would lower myrisk of having an accident.The reasoning in the driver’s argument is mostvulnerable to criticism on the grounds that thisargument(A)(B)(C)(D)(E)infers a cause from a mere correlationrelies on a sample that is too narrowmisinterprets evidence that a result is likely asevidence that the result is certainmistakes a condition sufficient for bringingabout a result for a condition necessary fordoing sorelies on a source that is probably notwell-informed23. Philosopher: An action is morally right if it would bereasonably expected to increase the aggregatewell-being of the people affected by it. An actionis morally wrong if and only if it would bereasonably expected to reduce the aggregate wellbeingof the people affected by it. Thus, actionsthat would be reasonably expected to leaveunchanged the aggregate well-being of the peopleaffected by them are also right.The philosopher’s conclusion follows logically if whichone of the following is assumed?(A)(B)(C)(D)(E)Only wrong actions would be reasonablyexpected to reduce the aggregate well-being ofthe people affected by them.No action is both right and wrong.Any action that is not morally wrong is morallyright.There are actions that would be reasonablyexpected to leave unchanged the aggregatewell-being of the people affected by them.Only right actions have good consequences.GO ON TO THE NEXT PAGE.


24. Car companies solicit consumer information on suchhuman factors as whether a seat is comfortable orwhether a set of controls is easy to use. However,designer interaction with consumers is superior tosurvey data; the data may tell the designer why afeature on last year’s model was given a low rating,but data will not explain how that feature needs to bechanged in order to receive a higher rating.The reasoning above conforms most closely to whichone of the following propositions?(A)(B)(C)(D)(E)Getting consumer input for design modificationscan contribute to successful product design.Car companies traditionally conduct extensivepostmarket surveys.Designers aim to create features that will appealto specific market niches.A car will have unappealing features ifconsumers are not consulted during its designstage.Consumer input affects external rather thaninternal design components of cars.www.cambridgelsat.com25. During the nineteenth century, the French academy ofart was a major financial sponsor of painting andsculpture in France; sponsorship by private individualshad decreased dramatically by this time. Because theacademy discouraged innovation in the arts, there waslittle innovation in nineteenth century French sculpture.Yet nineteenth century French painting showed aremarkable degree of innovation.2 2 2-13-2Which one of the following, if true, most helps toexplain the difference between the amount ofinnovation in French painting and the amount ofinnovation in French sculpture during the nineteenthcentury?(A)(B)(C)(D)(E)In France in the nineteenth century, the Frenchacademy gave more of its financial support topainting than it did to sculpture.The French academy in the nineteenth centuryfinancially supported a greater number ofsculptors than painters, but individual paintersreceived more support, on average, thanindividual sculptors.Because stone was so much more expensivethan paint and canvas, far more unsponsoredpaintings were produced than wereunsponsored sculptures in France during thenineteenth century.Very few of the artists in France in thenineteenth century who produced sculpturesalso produced paintings.Although the academy was the primary sponsorof sculpture and painting, the total amount offinancial support that French sculptors andpainters received from sponsors declinedduring the nineteenth century.S T O PIF YOU FINISH BEFORE TIME IS CALLED, YOU MAY CHECK YOUR WORK ON THIS SECTION ONLY.DO NOT WORK ON ANY OTHER SECTION IN THE TEST.13


3 -14- 14www.cambridgelsat.com3 3 3 3SECTION IIITime—35 minutes25 QuestionsDirections: The questions in this section are based on the reasoning contained in brief statements or passages. For somequestions, more than one of the choices could conceivably answer the question. However, you are to choose the best answer; thatis, the response that most accurately and completely answers the question. You should not make assumptions that are bycommonsense standards implausible, superfluous, or incompatible with the passage. After you have chosen the best answer,blacken the corresponding space on your answer sheet.1. Situation: Someone living in a cold climate buys awinter coat that is stylish but not warm in order toappear sophisticated.Analysis: People are sometimes willing to sacrificesensual comfort or pleasure for the sake ofappearances.The analysis provided for the situation above is mostappropriate for which one of the following situations?(A)(B)(C)(D)(E)A person buys an automobile to commute towork even though public transportation isquick and reliable.A parent buys a car seat for a young childbecause it is more colorful and morecomfortable for the child than the other carseats on the market, though no safer.A couple buys a particular wine even thoughtheir favorite wine is less expensive and bettertasting because they think it will impress theirdinner guests.A person sets her thermostat at a lowtemperature during the winter because she isconcerned about the environmental damagecaused by using fossil fuels to heat her home.An acrobat convinces the circus that employshim to purchase an expensive outfit for him sothat he can wear it during his act to impressthe audience.3. Carolyn: The artist Marc Quinn has displayed, behind aglass plate, biologically replicated fragments ofSir John Sulston’s DNA, calling it a “conceptualportrait” of Sulston. But to be a portrait, somethingmust bear a recognizable resemblance to itssubject.Arnold: I disagree. Quinn’s conceptual portrait is amaximally realistic portrait, for it holds actualinstructions according to which Sulston wascreated.The dialogue provides most support for the claim thatCarolyn and Arnold disagree over whether the objectdescribed by Quinn as a conceptual portrait of Sir JohnSulston(A)(B)(C)(D)(E)should be considered to be artshould be considered to be Quinn’s workbears a recognizable resemblance to Sulstoncontains instructions according to which Sulstonwas createdis actually a portrait of SulstonGO ON TO THE NEXT PAGE.2. After replacing his old gas water heater with a new,pilotless, gas water heater that is rated as highlyefficient, Jimmy’s gas bills increased.Each of the following, if true, contributes to anexplanation of the increase mentioned above EXCEPT:(A)(B)(C)(D)(E)The new water heater uses a smaller percentageof the gas used by Jimmy’s household than didthe old one.Shortly after the new water heater was installed,Jimmy’s uncle came to live with him, doublingthe size of the household.After having done his laundry at a laundromat,Jimmy bought and started using a gas dryerwhen he replaced his water heater.Jimmy’s utility company raised the rates for gasconsumption following installation of the newwater heater.Unusually cold weather following installation ofthe new water heater resulted in heavy gasusage.


4. Many corporations have begun decorating their hallswith motivational posters in hopes of boosting theiremployees’ motivation to work productively. However,almost all employees at these corporations are alreadymotivated to work productively. So these corporations’use of motivational posters is unlikely to achieve itsintended purpose.The reasoning in the argument is most vulnerable tocriticism on the grounds that the argument(A)(B)(C)(D)(E)fails to consider whether corporations that donot currently use motivational posters wouldincrease their employees’ motivation to workproductively if they began using the posterstakes for granted that, with respect to theiremployees’ motivation to work productively,corporations that decorate their halls withmotivational posters are representative ofcorporations in generalfails to consider that even if motivationalposters do not have one particular beneficialeffect for corporations, they may have similareffects that are equally beneficialdoes not adequately address the possibility thatemployee productivity is strongly affected byfactors other than employees’ motivation towork productivelyfails to consider that even if employees arealready motivated to work productively,motivational posters may increase thatmotivation5. Atrens: An early entomologist observed ants carryingparticles to neighboring ant colonies and inferredthat the ants were bringing food to theirneighbors. Further research, however, revealedthat the ants were emptying their own colony’sdumping site. Thus, the early entomologist waswrong.Atrens’s conclusion follows logically if which one ofthe following is assumed?(A)(B)(C)(D)(E)Ant societies do not interact in all the sameways that human societies interact.There is only weak evidence for the view thatants have the capacity to make use of objectsas gifts.Ant dumping sites do not contain particles thatcould be used as food.The ants to whom the particles were broughtnever carried the particles into their owncolonies.The entomologist cited retracted his conclusionwhen it was determined that the particles theants carried came from their dumping site.www.cambridgelsat.com3 3 3 3-15-36. Jablonski, who owns a car dealership, has donated carsto driver education programs at area schools for overfive years. She found the statistics on car accidents to bedisturbing, and she wanted to do something toencourage better driving in young drivers. Somemembers of the community have shown their support forthis action by purchasing cars from Jablonski’sdealership.Which one of the following propositions is bestillustrated by the passage?(A)(B)(C)(D)(E)The only way to reduce traffic accidents isthrough driver education programs.Altruistic actions sometimes have positiveconsequences for those who perform them.Young drivers are the group most likely tobenefit from driver education programs.It is usually in one’s best interest to performactions that benefit others.An action must have broad community supportif it is to be successful.7. Antonio: One can live a life of moderation by neverdeviating from the middle course. But then oneloses the joy of spontaneity and misses theopportunities that come to those who areoccasionally willing to take great chances, or togo too far.Marla: But one who, in the interests of moderation,never risks going too far is actually failing to livea life of moderation: one must be moderate evenin one’s moderation.Antonio and Marla disagree over(A)(B)(C)(D)(E)whether it is desirable for people occasionallyto take great chances in lifewhat a life of moderation requires of a personwhether it is possible for a person to embraceother virtues along with moderationhow often a person ought to deviate from themiddle course in lifewhether it is desirable for people to bemoderately spontaneousGO ON TO THE NEXT PAGE.15


3 -16- 168. Advertisement: Fabric-Soft leaves clothes soft andfluffy, and its fresh scent is a delight. Weconducted a test using over 100 consumers toprove Fabric-Soft is best. Each consumer wasgiven one towel washed with Fabric-Soft and onetowel washed without it. Ninety-nine percent ofthe consumers preferred the Fabric-Soft towel. SoFabric-Soft is the most effective fabric softeneravailable.The advertisement’s reasoning is most vulnerable tocriticism on the grounds that it fails to consider whether(A)(B)(C)(D)(E)any of the consumers tested are allergic tofabric softenersFabric-Soft is more or less harmful to theenvironment than other fabric softenersFabric-Soft is much cheaper or more expensivethan other fabric softenersthe consumers tested find the benefits of usingfabric softeners worth the expensethe consumers tested had the opportunity toevaluate fabric softeners other than Fabric-Soft9. Naturalist: The recent claims that the Tasmanian tiger isnot extinct are false. The Tasmanian tiger’snatural habitat was taken over by sheep farmingdecades ago, resulting in the animal’s systematicelimination from the area. Since then naturalistsworking in the region have discovered no hardevidence of its survival, such as carcasses ortracks. In spite of alleged sightings of the animal,the Tasmanian tiger no longer exists.Which one of the following is an assumption on whichthe naturalist’s argument depends?(A)(B)(C)(D)(E)Sheep farming drove the last Tasmanian tigersto starvation by chasing them from theirnatural habitat.Some scavengers in Tasmania are capable ofdestroying tiger carcasses without a trace.Every naturalist working in the Tasmaniantiger’s natural habitat has looked systematicallyfor evidence of the tiger’s survival.The Tasmanian tiger did not move and adapt toa different region in response to the loss ofhabitat.Those who have reported sightings of theTasmanian tiger are not experienced naturalists.www.cambridgelsat.com3 3 3 310. Advertisers have learned that people are more easilyencouraged to develop positive attitudes about thingstoward which they originally have neutral or evennegative attitudes if those things are linked, withpictorial help rather than exclusively through prose, tothings about which they already have positive attitudes.Therefore, advertisers are likely to _______.Which one of the following most logically completesthe argument?(A)(B)(C)(D)(E)use little if any written prose in theiradvertisementstry to encourage people to develop positiveattitudes about products that can be betterrepresented pictorially than in proseplace their advertisements on television ratherthan in magazineshighlight the desirable features of the advertisedproduct by contrasting them pictorially withundesirable features of a competing productcreate advertisements containing pictures ofthings most members of the target audiencelike11. Feathers recently taken from seabirds stuffed andpreserved in the 1880s have been found to contain onlyhalf as much mercury as feathers recently taken fromliving birds of the same species. Since mercury thataccumulates in a seabird’s feathers as the feathers growis derived from fish eaten by the bird, these resultsindicate that mercury levels in saltwater fish are highernow than they were 100 years ago.The argument depends on assuming that(A)(B)(C)(D)(E)the proportion of a seabird’s diet consisting offish was not as high, on average, in the 1880sas it is todaythe amount of mercury in a saltwater fishdepends on the amount of pollution in theocean habitat of the fishmercury derived from fish is essential for thenormal growth of a seabird’s feathersthe stuffed seabirds whose feathers were testedfor mercury were not fully grownthe process used to preserve birds in the 1880sdid not substantially decrease the amount ofmercury in the birds’ feathersGO ON TO THE NEXT PAGE.


12. Novel X and Novel Y are both semiautobiographicalnovels and contain many very similar themes andsituations, which might lead one to suspect plagiarismon the part of one of the authors. However, it is morelikely that the similarity of themes and situations in thetwo novels is merely coincidental, since both authors arefrom very similar backgrounds and have led similarlives.Which one of the following most accurately expressesthe conclusion drawn in the argument?(A)(B)(C)(D)(E)Novel X and Novel Y are bothsemiautobiographical novels, and the twonovels contain many very similar themes andsituations.The fact that Novel X and Novel Y are bothsemiautobiographical novels and contain manyvery similar themes and situations might leadone to suspect plagiarism on the part of one ofthe authors.The author of Novel X and the author ofNovel Y are from very similar backgroundsand have led very similar lives.It is less likely that one of the authors ofNovel X or Novel Y is guilty of plagiarismthan that the similarity of themes andsituations in the two novels is merelycoincidental.If the authors of Novel X and Novel Y are fromvery similar backgrounds and have led similarlives, suspicions that either of the authorsplagiarized are very likely to be unwarranted.www.cambridgelsat.com3 3 3 3-17-313. Therapist: Cognitive psychotherapy focuses onchanging a patient’s conscious beliefs. Thus,cognitive psychotherapy is likely to be moreeffective at helping patients overcomepsychological problems than are forms ofpsychotherapy that focus on changingunconscious beliefs and desires, since onlyconscious beliefs are under the patient’s directconscious control.Which one of the following, if true, would moststrengthen the therapist’s argument?(A)(B)(C)(D)(E)Psychological problems are frequently causedby unconscious beliefs that could be changedwith the aid of psychotherapy.It is difficult for any form of psychotherapy tobe effective without focusing on mental statesthat are under the patient’s direct consciouscontrol.Cognitive psychotherapy is the only form ofpsychotherapy that focuses primarily onchanging the patient’s conscious beliefs.No form of psychotherapy that focuses onchanging the patient’s unconscious beliefs anddesires can be effective unless it also helpschange beliefs that are under the patient’sdirect conscious control.All of a patient’s conscious beliefs are under thepatient’s conscious control, but otherpsychological states cannot be controlledeffectively without the aid of psychotherapy.17GO ON TO THE NEXT PAGE.


3 -18- 1814. Commentator: In academic scholarship, sources arealways cited, and methodology and theoreticalassumptions are set out, so as to allow criticalstudy, replication, and expansion of scholarship.In open-source software, the code in which theprogram is written can be viewed and modified byindividual users for their purposes without gettingpermission from the producer or paying a fee. Incontrast, the code of proprietary software is keptsecret, and modifications can be made only by theproducer, for a fee. This shows that open-sourcesoftware better matches the values embodied inacademic scholarship, and since scholarship iscentral to the mission of universities, universitiesshould use only open-source software.The commentator’s reasoning most closely conforms towhich one of the following principles?(A)(B)(C)(D)(E)Whatever software tools are most advanced andcan achieve the goals of academic scholarshipare the ones that should alone be used inuniversities.Universities should use the type of softwaretechnology that is least expensive, as long asthat type of software technology is adequatefor the purposes of academic scholarship.Universities should choose the type of softwaretechnology that best matches the valuesembodied in the activities that are central tothe mission of universities.The form of software technology that bestmatches the values embodied in the activitiesthat are central to the mission of universities isthe form of software technology that is mostefficient for universities to use.A university should not pursue any activity thatwould block the achievement of the goals ofacademic scholarship at that university.www.cambridgelsat.com3 3 3 315. A consumer magazine surveyed people who had soughta psychologist’s help with a personal problem. Of thoseresponding who had received treatment for 6 months orless, 20 percent claimed that treatment “made things alot better.” Of those responding who had received longertreatment, 36 percent claimed that treatment “madethings a lot better.” Therefore, psychological treatmentlasting more than 6 months is more effective thanshorter-term treatment.Which one of the following, if true, most seriouslyweakens the argument?(A)(B)(C)(D)(E)Of the respondents who had received treatmentfor longer than 6 months, 10 percent said thattreatment made things worse.Patients who had received treatment for longerthan 6 months were more likely to respond tothe survey than were those who had receivedtreatment for a shorter time.Patients who feel they are doing well intreatment tend to remain in treatment, whilethose who are doing poorly tend to quit earlier.Patients who were dissatisfied with theirtreatment were more likely to feel a need toexpress their feelings about it and thus toreturn the survey.Many psychologists encourage their patients toreceive treatment for longer than 6 months.16. Philosopher: Nations are not literally persons; theyhave no thoughts or feelings, and, literallyspeaking, they perform no actions. Thus theyhave no moral rights or responsibilities. But nonation can survive unless many of its citizensattribute such rights and responsibilities to it, fornothing else could prompt people to make thesacrifices national citizenship demands.Obviously, then, a nation _______.Which one of the following most logically completesthe philosopher’s argument?(A)(B)(C)(D)(E)cannot continue to exist unless something otherthan the false belief that the nation has moralrights motivates its citizens to make sacrificescannot survive unless many of its citizens havesome beliefs that are literally falsecan never be a target of moral praise or blameis not worth the sacrifices that its citizens makeon its behalfshould always be thought of in metaphoricalrather than literal termsGO ON TO THE NEXT PAGE.


17. When exercising the muscles in one’s back, it isimportant, in order to maintain a healthy back, toexercise the muscles on opposite sides of the spineequally. After all, balanced muscle development isneeded to maintain a healthy back, since the muscles onopposite sides of the spine must pull equally inopposing directions to keep the back in properalignment and protect the spine.Which one of the following is an assumption requiredby the argument?(A)(B)(C)(D)(E)Muscles on opposite sides of the spine that areequally well developed will be enough to keepthe back in proper alignment.Exercising the muscles on opposite sides of thespine unequally tends to lead to unbalancedmuscle development.Provided that one exercises the muscles onopposite sides of the spine equally, one willhave a generally healthy back.If the muscles on opposite sides of the spine areexercised unequally, one’s back will beirreparably damaged.One should exercise daily to ensure that themuscles on opposite sides of the spine keepthe back in proper alignment.www.cambridgelsat.com3 3 3 3-19-319. Editor: Many candidates say that if elected they willreduce governmental intrusion into voters’ lives.But voters actually elect politicians who insteadpromise that the government will provideassistance to solve their most pressing problems.Governmental assistance, however, costs money,and money can come only from taxes, which canbe considered a form of governmental intrusion.Thus, governmental intrusion into the lives ofvoters will rarely be substantially reduced overtime in a democracy.Which one of the following, if true, would moststrengthen the editor’s argument?(A)(B)(C)(D)(E)Politicians who win their elections usually keeptheir campaign promises.Politicians never promise what they reallyintend to do once in office.The most common problems people have arefinancial problems.Governmental intrusion into the lives of votersis no more burdensome in nondemocraticcountries than it is in democracies.Politicians who promise to do what theyactually believe ought to be done are rarelyelected.1918. Editorialist: In all cultures, it is almost universallyaccepted that one has a moral duty to preventmembers of one’s family from being harmed.Thus, few would deny that if a person is knownby the person’s parents to be falsely accused of acrime, it would be morally right for the parents tohide the accused from the police. Hence, it is alsolikely to be widely accepted that it is sometimesmorally right to obstruct the police in their work.The reasoning in the editorialist’s argument is mostvulnerable to criticism on the grounds that this argument(A)(B)(C)(D)(E)utilizes a single type of example for the purposeof justifying a broad generalizationfails to consider the possibility that other moralprinciples would be widely recognized asoverriding any obligation to protect a familymember from harmpresumes, without providing justification, thatallowing the police to arrest an innocentperson assists rather than obstructs justicetakes for granted that there is no moralobligation to obey the lawtakes for granted that the parents mentioned inthe example are not mistaken about theirchild’s innocenceGO ON TO THE NEXT PAGE.


3 -20- 2020. We should accept the proposal to demolish the old trainstation, because the local historical society, whichvehemently opposes this, is dominated by people whohave no commitment to long-term economic well-being.Preserving old buildings creates an impediment to newdevelopment, which is critical to economic health.The flawed reasoning exhibited by the argument aboveis most similar to that exhibited by which one of thefollowing arguments?(A)(B)(C)(D)(E)Our country should attempt to safeguard worksof art that it deems to possess national culturalsignificance. These works might not berecognized as such by all taxpayers, or evenall critics. Nevertheless, our country ought toexpend whatever money is needed to procureall such works as they become available.Documents of importance to local heritageshould be properly preserved and archived forthe sake of future generations. For, if even oneof these documents is damaged or lost, theintegrity of the historical record as a wholewill be damaged.You should have your hair cut no more thanonce a month. After all, beauticians suggestthat their customers have their hair cut twice amonth, and they do this as a way of generatingmore business for themselves.The committee should endorse the plan topostpone construction of the new expressway.Many residents of the neighborhoods thatwould be affected are fervently opposed to thatconstruction, and the committee is obligated toavoid alienating those residents.One should not borrow even small amounts ofmoney unless it is absolutely necessary. Onceone borrows a few dollars, the interest starts toaccumulate. The longer one takes to repay, themore one ends up owing, and eventually asmall debt has become a large one.www.cambridgelsat.com3 3 3 321. Ethicist: On average, animals raised on grain must befed sixteen pounds of grain to produce one poundof meat. A pound of meat is more nutritious forhumans than a pound of grain, but sixteen poundsof grain could feed many more people than coulda pound of meat. With grain yields leveling off,large areas of farmland going out of productioneach year, and the population rapidly expanding,we must accept the fact that consumption of meatwill soon be morally unacceptable.Which one of the following, if true, would most weakenthe ethicist’s argument?(A)(B)(C)(D)(E)Even though it has been established that avegetarian diet can be healthy, many peopleprefer to eat meat and are willing to pay for it.Often, cattle or sheep can be raised to maturityon grass from pastureland that is unsuitable forany other kind of farming.If a grain diet is supplemented with proteinderived from non-animal sources, it can havenutritional value equivalent to that of a dietcontaining meat.Although prime farmland near metropolitanareas is being lost rapidly to suburbandevelopment, we could reverse this trend bychoosing to live in areas that are alreadyurban.Nutritionists agree that a diet composed solelyof grain products is not adequate for humanhealth.GO ON TO THE NEXT PAGE.


22. If the price it pays for coffee beans continues toincrease, the Coffee Shoppe will have to increase itsprices. In that case, either the Coffee Shoppe will beginselling noncoffee products or its coffee sales willdecrease. But selling noncoffee products will decreasethe Coffee Shoppe’s overall profitability. Moreover, theCoffee Shoppe can avoid a decrease in overallprofitability only if its coffee sales do not decrease.Which one of the following statements follows logicallyfrom the statements above?(A)(B)(C)(D)(E)If the Coffee Shoppe’s overall profitabilitydecreases, the price it pays for coffee beanswill have continued to increase.If the Coffee Shoppe’s overall profitabilitydecreases, either it will have begun sellingnoncoffee products or its coffee sales will havedecreased.The Coffee Shoppe’s overall profitability willdecrease if the price it pays for coffee beanscontinues to increase.The price it pays for coffee beans cannotdecrease without the Coffee Shoppe’s overallprofitability also decreasing.Either the price it pays for coffee beans willcontinue to increase or the Coffee Shoppe’scoffee sales will increase.23. Political candidates’ speeches are loaded with promisesand with expressions of good intention, but one mustnot forget that the politicians’ purpose in giving thesespeeches is to get themselves elected. Clearly, then,these speeches are selfishly motivated and the promisesmade in them are unreliable.Which one of the following most accurately describes aflaw in the argument above?(A)(B)(C)(D)(E)The argument presumes, without providingjustification, that if a person’s promise is notselfishly motivated then that promise isreliable.The argument presumes, without providingjustification, that promises made for selfishreasons are never kept.The argument confuses the effect of an actionwith its cause.The argument overlooks the fact that a promiseneed not be unreliable just because the personwho made it had an ulterior motive for doing so.The argument overlooks the fact that acandidate who makes promises for selfishreasons may nonetheless be worthy of theoffice for which he or she is running.www.cambridgelsat.com3 3 3 3-21-324. Sociologist: Romantics who claim that people are notborn evil but may be made evil by the imperfectinstitutions that they form cannot be right, forthey misunderstand the causal relationshipbetween people and their institutions. After all,institutions are merely collections of people.Which one of the following principles, if valid, wouldmost help to justify the sociologist’s argument?(A)(B)(C)(D)(E)People acting together in institutions can domore good or evil than can people actingindividually.Institutions formed by people are inevitablyimperfect.People should not be overly optimistic in theirview of individual human beings.A society’s institutions are the surest gauge ofthat society’s values.The whole does not determine the properties ofthe things that compose it.25. Some anthropologists argue that the human speciescould not have survived prehistoric times if the specieshad not evolved the ability to cope with diverse naturalenvironments. However, there is considerable evidencethat Australopithecus afarensis, a prehistoric speciesrelated to early humans, also thrived in a diverse arrayof environments, but became extinct. Hence, theanthropologists’ claim is false.The reasoning in the argument is most vulnerable tocriticism on the grounds that the argument(A)(B)(C)(D)(E)confuses a condition’s being required for agiven result to occur in one case with thecondition’s being sufficient for such a result tooccur in a similar casetakes for granted that if one species had acharacteristic that happened to enable it tosurvive certain conditions, at least one relatedextinct species must have had the samecharacteristicgeneralizes, from the fact that one species witha certain characteristic survived certainconditions, that all related species with thesame characteristic must have survived exactlythe same conditionsfails to consider the possibility thatAustralopithecus afarensis had one or morecharacteristics that lessened its chances ofsurviving prehistoric timesfails to consider the possibility that, even if acondition caused a result to occur in one case,it was not necessary to cause the result tooccur in a similar caseS T O PIF YOU FINISH BEFORE TIME IS CALLED, YOU MAY CHECK YOUR WORK ON THIS SECTION ONLY.DO NOT WORK ON ANY OTHER SECTION IN THE TEST.21


4 -22- 22www.cambridgelsat.com  SECTION IV4 4 4 4 4Time—35 minutes27 QuestionsDirections: Each set of questions in this section is based on a single passage or a pair of passages. The questions are to beanswered on the basis of what is stated or implied in the passage or pair of passages. For some of the questions, more than oneof the choices could conceivably answer the question. However, you are to choose the best answer; that is, the response thatmost accurately and completely answers the question, and blacken the corresponding space on your answer sheet.For decades, there has been a deep rift betweenpoetry and fiction in the United States, especially inacademic settings; graduate writing programs inuniversities, for example, train students as poets or as(5) writers of fiction, but almost never as both. Both poetsand writers of fiction have tended to support thisseparation, in large part because the currentconventional wisdom holds that poetry should beelliptical and lyrical, reflecting inner states and(10) processes of thought or feeling, whereas character andnarrative events are the stock-in-trade of fiction.Certainly it is true that poetry and fiction aredistinct genres, but why have specialized educationand literary territoriality resulted from this distinction?(15) The answer lies perhaps in a widespread attitude inU.S. culture, which often casts a suspicious eye on thegeneralist. Those with knowledge and expertise inmultiple areas risk charges of dilettantism, as if abilityin one field is diluted or compromised by(20) accomplishment in another.Fortunately, there are signs that the bias againstwriters who cross generic boundaries is diminishing;several recent writers are known and respected fortheir work in both genres. One important example of(25) this trend is Rita Dove, an African American writerhighly acclaimed for both her poetry and her fiction. Afew years ago, speaking at a conference entitled “PoetsWho Write Fiction,” Dove expressed gentleincredulity about the habit of segregating the genres.(30) She had grown up reading and loving both fiction andpoetry, she said, unaware of any purported dangerlurking in attempts to mix the two. She also studied forsome time in Germany, where, she observes, “Poetswrite plays, novelists compose libretti, playwrights(35) write novels—they would not understand ourrestrictiveness.”It makes little sense, Dove believes, to persist inthe restrictive approach to poetry and fiction prevalentin the U.S., because each genre shares in the nature of(40) the other. Indeed, her poetry offers example afterexample of what can only be properly regarded aslyric narrative. Her use of language in these poems isundeniably lyrical—that is, it evokes emotion andinner states without requiring the reader to organize(45) ideas or events in a particular linear structure. Yet thislyric expression simultaneously presents the elementsof a plot in such a way that the reader is led repeatedlyto take account of clusters of narrative details withinthe lyric flow. Thus while the language is lyrical, it(50) often comes to constitute, cumulatively, a work ofnarrative fiction. Similarly, many passages in herfiction, though undeniably prose, achieve the status oflyric narrative through the use of poetic rhythms andelliptical expression. In short, Dove bridges the gap(55) between poetry and fiction not only by writing in bothgenres, but also by fusing the two genres withinindividual works.1. Which one of the following most accurately expressesthe main point of the passage?(A)(B)(C)(D)(E)Rita Dove’s work has been widely acclaimedprimarily because of the lyrical elements shehas introduced into her fiction.Rita Dove’s lyric narratives present clusters ofnarrative detail in order to create a cumulativenarrative without requiring the reader tointerpret it in a linear manner.Working against a bias that has long beendominant in the U.S., recent writers like RitaDove have shown that the lyrical use oflanguage can effectively enhance narrativefiction.Unlike many of her U.S. contemporaries, RitaDove writes without relying on the traditionaltechniques associated with poetry and fiction.Rita Dove’s successful blending of poetry andfiction exemplifies the recent trend away fromthe rigid separation of the two genres that haslong been prevalent in the U.S.2. Which one of the following is most analogous to theliterary achievements that the author attributes to Dove?(A)(B)(C)(D)(E)A chef combines nontraditional cookingmethods and traditional ingredients fromdisparate world cuisines to devise new recipes.A professor of film studies becomes a filmdirector and succeeds, partly due to a wealthof theoretical knowledge of filmmaking.An actor who is also a theatrical director teamsup with a public health agency to use streettheater to inform the public about healthmatters.A choreographer defies convention andchoreographs dances that combine elements ofboth ballet and jazz dance.A rock musician records several songs fromprevious decades but introduces extendedguitar solos into each one.GO ON TO THE NEXT PAGE.


3. According to the passage, in the U.S. there is a widelyheld view that(A)(B)(C)(D)(E)poetry should not involve characters ornarrativesunlike the writing of poetry, the writing offiction is rarely an academically seriousendeavorgraduate writing programs focus on poetry tothe exclusion of fictionfiction is most aesthetically effective when itincorporates lyrical elementsEuropean literary cultures are suspicious ofgeneralists4. The author’s attitude toward the deep rift betweenpoetry and fiction in the U.S. can be most accuratelydescribed as one of(A)(B)(C)(D)(E)perplexity as to what could have led to thedevelopment of such a riftastonishment that academics have overlookedthe existence of the riftambivalence toward the effect the rift has hadon U.S. literaturepessimism regarding the possibility that the riftcan be overcomedisapproval of attitudes and presuppositionsunderlying the rift5. In the passage the author conjectures that a cause of thedeep rift between fiction and poetry in the United Statesmay be that(A)(B)(C)(D)(E)poets and fiction writers each tend to see theircraft as superior to the others’ craftthe methods used in training graduate studentsin poetry are different from those used intraining graduate students in other literaryfieldspublishers often pressure writers to concentrateon what they do besta suspicion of generalism deters writers fromdividing their energies between the two genresfiction is more widely read and respected thanpoetrywww.cambridgelsat.com  4 4 4 4 4-23-46. In the context of the passage, the author’s primarypurpose in mentioning Dove’s experience in Germany(lines 32–36) is to(A)(B)(C)(D)(E)suggest that the habit of treating poetry andfiction as nonoverlapping domains ischaracteristic of English-speaking societies butnot otherspoint to an experience that reinforced Dove’sconviction that poetry and fiction should notbe rigidly separatedindicate that Dove’s strengths as a writer derivein large part from the international character ofher academic backgroundpresent an illuminating biographical detail aboutDove in an effort to enhance the humaninterest appeal of the passageindicate what Dove believes to be the origin ofher opposition to the separation of fiction andpoetry in the U.S.7. It can be inferred from the passage that the authorwould be most likely to believe which one of thefollowing?(A)(B)(C)(D)(E)Each of Dove’s works can be classified aseither primarily poetry or primarily fiction,even though it may contain elements of both.The aesthetic value of lyric narrative resides inits representation of a sequence of events,rather than in its ability to evoke inner states.The way in which Dove blends genres in herwriting is without precedent in U.S. writing.Narrative that uses lyrical language is generallyaesthetically superior to pure lyric poetry.Writers who successfully cross the genericboundary between poetry and fiction often trytheir hand at genres such as drama as well.8. If this passage had been excerpted from a longer text,which one of the following predictions about the nearfuture of U.S. literature would be most likely to appearin that text?(A)(B)(C)(D)(E)The number of writers who write both poetryand fiction will probably continue to grow.Because of the increased interest in mixedgenres, the small market for pure lyric poetrywill likely shrink even further.Narrative poetry will probably come to beregarded as a sub-genre of fiction.There will probably be a rise in specializationamong writers in university writing programs.Writers who continue to work exclusively inpoetry or fiction will likely lose theiraudiences.23GO ON TO THE NEXT PAGE.


www.cambridgelsat.com 4 24The two passages discuss recent scientific research onmusic. They are adapted from two different paperspresented at a scholarly conference.(55) prematurely, leaving them much more helpless at birth.This helplessness necessitated longer, better maternalcare. Under such conditions, the emotional bondsPassage Acreated in the premusical mother-infant interactions weobserve in Homo sapiens today—behavior whoseDid music and human language originateseparately or together? Both systems use intonationand rhythm to communicate emotions. Both can be(60) neurological basis essentially constitutes the capacityto make and enjoy music—would have conferredconsiderable evolutionary advantage.produced vocally or with tools, and people can produce(5) both music and language silently to themselves.Brain imaging studies suggest that music and9. Both passages were written primarily in order to answerwhich one of the following questions?language are part of one large, vastly complicated,neurological system for processing sound. In fact,(A) What evolutionary advantage did larger brainfewer differences than similarities exist between thesize confer on early hominids?(10) neurological processing of the two. One could think of(B) Why do human mothers and infants engage inthe two activities as different radio programs that canbonding behavior that is composed of musicalbe broadcast over the same hardware. One noteworthyelements?difference, though, is that, generally speaking, people(C) What are the evolutionary origins of the humanare better at language than music. In music, anyoneability to make music?(15) can listen easily enough, but most people do not(D) Do the human abilities to make music and toperform well, and in many cultures composition is leftuse language depend on the same neurologicalto specialists. In language, by contrast, nearlysystems?everyone actively performs and composes.(E) Why are most people more adept at usingGiven their shared neurological basis, it appearslanguage than they are at making music?(20) that music and language evolved together as brain sizeincreased over the course of hominid evolution. But10. Each of the two passages mentions the relation of musicthe primacy of language over music that we canto(25)observe today suggests that language, not music, was(A) bonding between humansthe primary function natural selection operated on.(B) human emotionMusic, it would seem, had little adaptive value of its(C) neurological researchown, and most likely developed on the coattails of(D) the increasing helplessness of hominid infantslanguage.(E) the use of tools to produce soundsPassage BDarwin claimed that since “neither the enjoymentnor the capacity of producing musical notes are(30) faculties of the least [practical] use to manthey mustbe ranked amongst the most mysterious with which heGO ON TO THE NEXT PAGE.is endowed.” I suggest that the enjoyment of and thecapacity to produce musical notes are facultiesof indispensable use to mothers and their infants and(35) that it is in the emotional bonds created by theinteraction of mother and child that we can discoverthe evolutionary origins of human music.Even excluding lullabies, which parents sing toinfants, human mothers and infants under six months(40) of age engage in ritualized, sequential behaviors,involving vocal, facial, and bodily interactions. Usingface-to-face mother-infant interactions filmed at 24frames per second, researchers have shown thatmothers and infants jointly construct mutually(45) improvised interactions in which each partner tracksthe actions of the other. Such episodes last fromone-half second to three seconds and are composed ofmusical elements—variations in pitch, rhythm, timbre,volume, and tempo.(50) What evolutionary advantage would suchbehavior have? In the course of hominid evolution,brain size increased rapidly. Contemporaneously, theincrease in bipedality caused the birth canal to narrow.This resulted in hominid infants being born ever-more4 4 4 4 4


www.cambridgelsat.com  11. It can be inferred that the authors of the two passageswould be most likely to disagree over whether13. Which one of the following principles underlies thearguments in both passages?4 4 4 4 4-25-4(A)(B)(C)(D)(E)the increase in hominid brain size necessitatedearlier birthsfewer differences than similarities exist betweenthe neurological processing of music andhuman languagebrain size increased rapidly over the course ofhuman evolutionthe capacity to produce music has greatadaptive value to humansmother-infant bonding involves temporallypatterned vocal interactions12. The authors would be most likely to agree on theanswer to which one of the following questionsregarding musical capacity in humans?(A)(B)(C)(D)(E)Does it manifest itself in some form in earlyinfancy?Does it affect the strength of mother-infantbonds?Is it at least partly a result of evolutionaryincreases in brain size?Did its evolution spur the development of newneurological systems?Why does it vary so greatly among differentindividuals?(A)(B)(C)(D)(E)Investigations of the evolutionary origins ofhuman behaviors must take into account thebehavior of nonhuman animals.All human capacities can be explained in termsof the evolutionary advantages they offer.The fact that a single neurological systemunderlies two different capacities is evidencethat those capacities evolved concurrently.The discovery of the neurological basis of ahuman behavior constitutes the discovery ofthe essence of that behavior.The behavior of modern-day humans canprovide legitimate evidence concerning theevolutionary origins of human abilities.14. Which one of the following most accuratelycharacterizes a relationship between the two passages?(A)(B)(C)(D)(E)Passage A and passage B use different evidenceto draw divergent conclusions.Passage A poses the question that passage Battempts to answer.Passage A proposes a hypothesis that passage Battempts to substantiate with new evidence.Passage A expresses a stronger commitment toits hypothesis than does passage B.Passage A and passage B use different evidenceto support the same conclusion.25GO ON TO THE NEXT PAGE.


4 -26- 26The World Wide Web, a network of electronicallyproduced and interconnected (or “linked”) sites, calledpages, that are accessible via personal computer, raiseslegal issues about the rights of owners of intellectual(5) property, notably those who create documents forinclusion on Web pages. Some of these owners ofintellectual property claim that unless copyright law isstrengthened, intellectual property on the Web will notbe protected from copyright infringement. Web users,(10) however, claim that if their ability to accessinformation on Web pages is reduced, the Web cannotlive up to its potential as an open, interactive mediumof communication.The debate arises from the Web’s ability to link(15) one document to another. Links between sites areanalogous to the inclusion in a printed text ofreferences to other works, but with one difference: thecited document is instantly retrievable by a user whoactivates the link. This immediate accessibility creates(20) a problem, since current copyright laws give owners ofintellectual property the right to sue a distributor ofunauthorized copies of their material even if thatdistributor did not personally make the copies. Ifperson A, the author of a document, puts the document(25) on a Web page, and person B, the creator of anotherWeb page, creates a link to A’s document, is Bcommitting copyright infringement?To answer this question, it must first bedetermined who controls distribution of a document on(30) the Web. When A places a document on a Web page,this is comparable to recording an outgoing messageon one’s telephone answering machine for others tohear. When B creates a link to A’s document, this isakin to B’s giving out A’s telephone number, thereby(35) allowing third parties to hear the outgoing message forthemselves. Anyone who calls can listen to themessage; that is its purpose. While B’s link mayindeed facilitate access to A’s document, the crucialpoint is that A, simply by placing that document on the(40) Web, is thereby offering it for distribution. Therefore,even if B leads others to the document, it is A whoactually controls access to it. Hence creating a link to adocument is not the same as making or distributing acopy of that document. Moreover, techniques are(45) already available by which A can restrict access to adocument. For example, A may require a password togain entry to A’s Web page, just as a telephone ownercan request an unlisted number and disclose it only toselected parties. Such a solution would compromise(50) the openness of the Web somewhat, but not as much asthe threat of copyright infringement litigation.Changing copyright law to benefit owners ofintellectual property is thus ill-advised because itwould impede the development of the Web as a public(55) forum dedicated to the free exchange of ideas.www.cambridgelsat.com 4 4 4 4 415. Which one of the following most accurately expressesthe main point of the passage?(A)(B)(C)(D)(E)Since distribution of a document placed on aWeb page is controlled by the author of thatpage rather than by the person who creates alink to the page, creating such a link shouldnot be considered copyright infringement.Changes in copyright law in response to thedevelopment of Web pages and links areill-advised unless such changes amplify ratherthan restrict the free exchange of ideasnecessary in a democracy.People who are concerned about the accessothers may have to the Web documents theycreate can easily prevent such access withoutinhibiting the rights of others to exchangeideas freely.Problems concerning intellectual property rightscreated by new forms of electronic media arenot insuperably difficult to resolve if oneapplies basic commonsense principles to theseproblems.Maintaining a free exchange of ideas on theWeb offers benefits that far outweigh thosethat might be gained by a small number ofindividuals if a radical alteration of copyrightlaws aimed at restricting the Web’s growthwere allowed.16. Which one of the following is closest in meaning to theterm “strengthened” as that term is used in line 8 of thepassage?(A)(B)(C)(D)(E)made more restrictivemade uniform worldwidemade to impose harsher penaltiesdutifully enforcedmore fully recognized as legitimateGO ON TO THE NEXT PAGE.


17. With which one of the following claims aboutdocuments placed on Web pages would the author bemost likely to agree?(A)(B)(C)(D)(E)Such documents cannot receive adequateprotection unless current copyright laws arestrengthened.Such documents cannot be protected fromunauthorized distribution without significantlydiminishing the potential of the Web to be awidely used form of communication.The nearly instantaneous access afforded by theWeb makes it impossible in practice to limitaccess to such documents.Such documents can be protected fromcopyright infringement with the least damageto the public interest only by altering existinglegal codes.Such documents cannot fully contribute to theWeb’s free exchange of ideas unless theirauthors allow them to be freely accessed bythose who wish to do so.18. Based on the passage, the relationship betweenstrengthening current copyright laws and relying onpasswords to restrict access to a Web document is mostanalogous to the relationship between(A)(B)(C)(D)(E)allowing everyone use of a public facility andrestricting its use to members of thecommunityoutlawing the use of a drug and outlawing itssaleprohibiting a sport and relying on participants toemploy proper safety gearpassing a new law and enforcing that lawallowing unrestricted entry to a building andrestricting entry to those who have been issueda badge19. The passage most strongly implies which one of thefollowing?(A)(B)(C)(D)(E)There are no creators of links to Web pageswho are also owners of intellectual property onWeb pages.The person who controls access to a Web pagedocument should be considered the distributorof that document.Rights of privacy should not be extended toowners of intellectual property placed on theWeb.Those who create links to Web pages haveprimary control over who reads the documentson those pages.A document on a Web page must be convertedto a physical document via printing beforecopyright infringement takes place.www.cambridgelsat.com  4 4 4 4 4-27-420. According to the passage, which one of the followingfeatures of outgoing messages left on telephoneanswering machines is most relevant to the debateconcerning copyright infringement?(A)(B)(C)(D)(E)Such messages are carried by an electronicmedium of communication.Such messages are not legally protected againstunauthorized distribution.Transmission of such messages is virtuallyinstantaneous.People do not usually care whether or notothers might record such messages.Such messages have purposely been madeavailable to anyone who calls that telephonenumber.21. The author’s discussion of telephone answeringmachines serves primarily to(A)(B)(C)(D)(E)compare and contrast the legal problems createdby two different sorts of electronic mediaprovide an analogy to illustrate the positionstaken by each of the two sides in the copyrightdebateshow that the legal problems produced by newcommunication technology are not themselvesnewillustrate the basic principle the author believesshould help determine the outcome of thecopyright debateshow that telephone use also raises concernsabout copyright infringement22. According to the passage, present copyright laws(A)(B)(C)(D)(E)allow completely unrestricted use of anydocument placed by its author on a Web pageallow those who establish links to a documenton a Web page to control its distribution toothersprohibit anyone but the author of a documentfrom making a profit from the document’sdistributionallow the author of a document to sue anyonewho distributes the document withoutpermissionshould be altered to allow more completefreedom in the exchange of ideasGO ON TO THE NEXT PAGE.27


4 -28- 28In tracing the changing face of the Irishlandscape, scholars have traditionally relied primarilyon evidence from historical documents. However, suchdocumentary sources provide a fragmentary record at(5) best. Reliable accounts are very scarce for many partsof Ireland prior to the seventeenth century, and manyof the relevant documents from the sixteenth andseventeenth centuries focus selectively on mattersrelating to military or commercial interests.(10) Studies of fossilized pollen grains preserved inpeats and lake muds provide an additional means ofinvestigating vegetative landscape change. Details ofchanges in vegetation resulting from both humanactivities and natural events are reflected in the kinds(15) and quantities of minute pollen grains that becometrapped in sediments. Analysis of samples can identifywhich kinds of plants produced the preserved pollengrains and when they were deposited, and in manycases the findings can serve to supplement or correct(20) the documentary record.For example, analyses of samples from LongLough in County Down have revealed significantpatterns of cereal-grain pollen beginning by about 400A.D. The substantial clay content of the soil in this part(25) of Down makes cultivation by primitive tools difficult.Historians thought that such soils were not tilled toany significant extent until the introduction of themoldboard plough to Ireland in the seventh centuryA.D. Because cereal cultivation would have required(30) tilling of the soil, the pollen evidence indicates thatthese soils must indeed have been successfully tilledbefore the introduction of the new plough.Another example concerns flax cultivation inCounty Down, one of the great linen-producing areas(35) of Ireland during the eighteenth century. Some aspectsof linen production in Down are well documented, butthe documentary record tells little about the cultivationof flax, the plant from which linen is made, in thatarea. The record of eighteenth-century linen(40) production in Down, together with the knowledge thatflax cultivation had been established in Irelandcenturies before that time, led some historians tosurmise that this plant was being cultivated in Downbefore the eighteenth century. But pollen analyses(45) indicate that this is not the case; flax pollen was foundonly in deposits laid down since the eighteenthcentury.It must be stressed, though, that there are limits tothe ability of the pollen record to reflect the vegetative(50) history of the landscape. For example, pollen analysescannot identify the species, but only the genus orfamily, of some plants. Among these is madder, acultivated dye plant of historical importance in Ireland.Madder belongs to a plant family that also comprises(55) various native weeds, including goosegrass. If madderpollen were present in a deposit it would beindistinguishable from that of uncultivated nativespecies.www.cambridgelsat.com 4 4 4 4 423. Which one of the following most accurately expressesthe main point of the passage?(A)(B)(C)(D)(E)Analysis of fossilized pollen is a useful meansof supplementing and in some cases correctingother sources of information regarding changesin the Irish landscape.Analyses of historical documents, together withpollen evidence, have led to the revision ofsome previously accepted hypotheses regardingchanges in the Irish landscape.Analysis of fossilized pollen has proven to be avaluable tool in the identification of ancientplant species.Analysis of fossilized pollen has provided newevidence that the cultivation of such crops ascereal grains, flax, and madder had asignificant impact on the landscape of Ireland.While pollen evidence can sometimessupplement other sources of historicalinformation, its applicability is severelylimited, since it cannot be used to identifyplant species.24. The passage indicates that pollen analyses have providedevidence against which one of the following views?(A)(B)(C)(D)(E)The moldboard plough was introduced intoIreland in the seventh century.In certain parts of County Down, cereal grainswere not cultivated to any significant extentbefore the seventh century.In certain parts of Ireland, cereal grains havebeen cultivated continuously since theintroduction of the moldboard plough.Cereal grain cultivation requires successfultilling of the soil.Cereal grain cultivation began in County Downaround 400 A.D.25. The phrase “documentary record” (lines 20 and 37)primarily refers to(A)(B)(C)(D)(E)documented results of analyses of fossilizedpollenthe kinds and quantities of fossilized pollengrains preserved in peats and lake mudswritten and pictorial descriptions by currenthistorians of the events and landscapes of pastcenturiesgovernment and commercial records, maps, andsimilar documents produced in the past thatrecorded conditions and events of that timearticles, books, and other documents by currenthistorians listing and analyzing all theavailable evidence regarding a particularhistorical periodGO ON TO THE NEXT PAGE.


26. The passage indicates that prior to the use of pollenanalysis in the study of the history of the Irishlandscape, at least some historians believed which oneof the following?(A)(B)(C)(D)(E)The Irish landscape had experienced significantflooding during the seventeenth century.Cereal grain was not cultivated anywhere inIreland until at least the seventh century.The history of the Irish landscape during thesixteenth and seventeenth centuries was welldocumented.Madder was not used as a dye plant in Irelanduntil after the eighteenth century.The beginning of flax cultivation in CountyDown may well have occurred before theeighteenth century.www.cambridgelsat.com  4 4 4 4 4-29-427. Which one of the following most accurately describesthe relationship between the second paragraph and thefinal paragraph?(A)(B)(C)(D)(E)The second paragraph proposes a hypothesis forwhich the final paragraph offers a supportingexample.The final paragraph describes a problem thatmust be solved before the method advocated inthe second paragraph can be considered viable.The final paragraph qualifies the claim made inthe second paragraph.The second paragraph describes a view againstwhich the author intends to argue, and the finalparagraph states the author’s argument againstthat view.The final paragraph offers procedures tosupplement the method described in the secondparagraph.S T O PIF YOU FINISH BEFORE TIME IS CALLED, YOU MAY CHECK YOUR WORK ON THIS SECTION ONLY.DO NOT WORK ON ANY OTHER SECTION IN THE TEST.29


Acknowledgmentswww.cambridgelsat.com3030Acknowledgment is made to the following sources from which material has been adapted for use in thistest booklet:Valerie A. Hall, “The Development of the Landscape of Ireland over the Last Two Thousand Years; Fresh Evidence from Historicaland Pollen Analytical Studies.” ©1997 by Chronicon, UCC.Timothy Miller, How to Want What You Have. ©1995 by Timothy Miller.Carol Muske, “Breaking Out of the Genre Ghetto.” ©1995 by Poetry in Review Foundation.


123456789012345678901234567890123456789012345678901234567890123456789012345678901234567890INSTRUCTIONS FOR COMPLETING THE BIOGRAPHICAL AREA ARE ON THE BACK COVER OF YOUR TEST BOOKLET.USE ONLY A NO. 2 OR HB PENCIL TO COMPLETE THIS ANSWER SHEET. DO NOT USE INK.SIDE 1LAST NAME FIRST NAME MIOQHGFEDCBAIJKLMNPRSTUVWXYZOQHGFEDCBAIJKLMNPRSTUVWXYZOQHGFEDCBAIJKLMNPRSTUVWXYZOQHGFEDCBAIJKLMNPRSTUVWXYZOQHGFEDCBAIJKLMNPRSTUVWXYZOQHGFEDCBAIJKLMNPRSTUVWXYZOQHGFEDCBAIJKLMNPRSTUVWXYZOQHGFEDCBAIJKLMNPRSTUVWXYZOQHGFEDCBAIJKLMNPRSTUVWXYZOQHGFEDCBAIJKLMNPRSTUVWXYZOQHGFEDCBAIJKLMNPRSTUVWXYZOQHGFEDCBAIJKLMNPRSTUVWXYZOQHGFEDCBAIJKLMNPRSTUVWXYZOQHGFEDCBAIJKLMNPRSTUVWXYZOQHGFEDCBAIJKLMNPRSTUVWXYZOQHGFEDCBAIJKLMNPRSTUVWXYZOQHGFEDCBAIJKLMNPRSTUVWXYZOQHGFEDCBAIJKLMNPRSTUVWXYZOQHGFEDCBAIJKLMNPRSTUVWXYZOQHGFEDCBAIJKLMNPRSTUVWXYZOQHGFEDCBAIJKLMNPRSTUVWXYZ2 LAST 4 DIGITS OFSOCIAL SECURITY/SOCIAL INSURANCE NO.ARACIAL/ETHNICDESCRIPTIONMark one or more1 Amer. Indian/Alaska Native2 Asian3 Black/African American4 Canadian Aboriginal5 Caucasian/White6 Hispanic/Latino7 Native Hawaiian/Other Pacific Islander8 Puerto RicanTEST FORMTEST DATEMONTH DAY YEARPLEASE PRINT INFORMATIONLAST NAMEFIRST NAMELaw School Admission TestMark one and only one answer to each question. Be sure to fill in completely the space foryour intended answer choice. If you erase, do so completely. Make no stray marks.123456789101112131415161718192021222324252627282930SECTION 1123456789101112131415161718192021222324252627282930SECTION 2123456789101112131415161718192021222324252627282930SECTION 3123456789101112131415161718192021222324252627282930SECTION 4123456789101112131415161718192021222324252627282930SECTION 5© 2013 BY LAW SCHOOL ADMISSION COUNCIL.ALL RIGHTS RESERVED. PRINTED IN USA.DATE OF BIRTHEDCBAEDCBAEDCBAEDCBAEDCBAEDCBAEDCBAEDCBAEDCBAEDCBAEDCBAEDCBAEDCBAEDCBAEDCBAEDCBAEDCBAEDCBAEDCBAEDCBAEDCBAEDCBAEDCBAEDCBAEDCBAEDCBAEDCBAEDCBAEDCBAEDCBAEDCBAEDCBAEDCBAEDCBAEDCBAEDCBAEDCBAEDCBAEDCBAEDCBAEDCBAEDCBAEDCBAEDCBAEDCBAEDCBAEDCBAEDCBAEDCBAEDCBAEDCBAEDCBAEDCBAEDCBAEDCBAEDCBAEDCBAEDCBAEDCBAEDCBAEDCBAEDCBAEDCBAEDCBAEDCBAEDCBAEDCBAEDCBAEDCBAEDCBAEDCBAEDCBAEDCBAEDCBAEDCBAEDCBAEDCBAEDCBAEDCBAEDCBAEDCBAEDCBAEDCBAEDCBAEDCBAEDCBAEDCBAEDCBAEDCBAEDCBAEDCBAEDCBAEDCBAEDCBAEDCBAEDCBAEDCBAEDCBAEDCBAEDCBAEDCBAEDCBAEDCBAEDCBAEDCBAEDCBAEDCBAEDCBAEDCBAEDCBAEDCBAEDCBAEDCBAEDCBAEDCBAEDCBAEDCBAEDCBAEDCBAEDCBAEDCBAEDCBAEDCBAEDCBAEDCBAEDCBAEDCBAEDCBAEDCBAEDCBAEDCBAEDCBAEDCBAEDCBAEDCBAEDCBAEDCBAEDCBAEDCBAEDCBAEDCBAEDCBAEDCBAEDCBAEDCBAEDCBAEDCBAEDCBAEDCBAEDCBAMONTHDAY YEARJanFebMarAprMay<strong>June</strong>JulyAugSeptOctNovDecDATE OF BIRTH1230123456789012345678901234567890LSACACCOUNT NUMBER12345678901234567890123456789012345678901234567890123456789012345678901234567890LBAGENDERMaleDOMINANTLANGUAGEEnglishOtherENGLISHFLUENCYNoYesFemaleEliteView EM-290725-3:654321CENTERNUMBER1234567890123456789012345678901234567890TESTFORM CODE12345678901234567890123456789012345678901234567890123456789012345678909 TSI/Aboriginal AustralianTEST BOOKSERIAL NO.123456789012345678901234567890123456789012345678901234567890S C A N T R O N®1 3 45 6 7121110981314OQHGFEDCBAIJKLMNPRSTOQHGFEDCBAIJKLMNPRSTwww.cambridgelsat.com


SIDE 2www.cambridgelsat.comGeneral Directions for the <strong>LSAT</strong> Answer SheetThe actual testing time for this portion of the test will be 2 hours 55 minutes. There are five sections, each with a time limit of 35minutes. The supervisor will tell you when to begin and end each section. If you finish a section before time is called, you may checkyour work on that section only; do not turn to any other section of the test book and do not work on any other section either in the testbook or on the answer sheet.There are several different types of questions on the test, and each question type has its own directions. Be sure you understand thedirections for each question type before attempting to answer any questions in that section.Not everyone will finish all the questions in the time allowed. Do not hurry, but work steadily and as quickly as you can withoutsacrificing accuracy. You are advised to use your time effectively. If a question seems too difficult, go on to the next one and return tothe difficult question after completing the section. MARK THE BEST ANSWER YOU CAN FOR EVERY QUESTION. NO DEDUCTIONSWILL BE MADE FOR WRONG ANSWERS. YOUR SCORE WILL BE BASED ONLY ON THE NUMBER OF QUESTIONS YOUANSWER CORRECTLY.ALL YOUR ANSWERS MUST BE MARKED ON THE ANSWER SHEET. Answer spaces for each question are lettered to correspondwith the letters of the potential answers to each question in the test book. After you have decided which of the answers is correct,blacken the corresponding space on the answer sheet. BE SURE THAT EACH MARK IS BLACK AND COMPLETELY FILLS THEANSWER SPACE. Give only one answer to each question. If you change an answer, be sure that all previous marks are erasedcompletely. Since the answer sheet is machine scored, incomplete erasures may be interpreted as intended answers. ANSWERSRECORDED IN THE TEST BOOK WILL NOT BE SCORED.There may be more question numbers on this answer sheet than there are questions in a section. Do not be concerned, but be certainthat the section and number of the question you are answering matches the answer sheet section and question number. Additionalanswer spaces in any answer sheet section should be left blank. Begin your next section in the number one answer space for thatsection.LSAC takes various steps to ensure that answer sheets are returned from test centers in a timely manner for processing. In the unlikelyevent that an answer sheet is not received, LSAC will permit the examinee either to retest at no additional fee or to receive a refund ofhis or her <strong>LSAT</strong> fee. THESE REMEDIES ARE THE ONLY REMEDIES AVAILABLE IN THE UNLIKELY EVENT THAT AN ANSWERSHEET IS NOT RECEIVED BY LSAC.Score CancellationComplete this section only if you areabsolutely certain you want to cancel yourscore. A CANCELLATION REQUESTCANNOT BE RESCINDED. IF YOU ARE ATALL UNCERTAIN, YOU SHOULD NOTCOMPLETE THIS SECTION.To cancel your score from thisadministration, you must:A. fill in both ovals here ......ANDB. read the following statement.Then sign your name and enterthe date.YOUR SIGNATURE ALONE ISNOT SUFFICIENT FOR SCORECANCELLATION. BOTHOVALS ABOVE MUST BEFILLED IN FOR SCANNINGEQUIPMENT TO RECOGNIZEYOUR REQUEST FOR SCORECANCELLATION.I certify that I wish to cancel my testscore from this administration. Iunderstand that my request isirreversible and that my score willnot be sent to me or to the lawschools to which I apply.HOW DID YOU PREPARE FOR THE <strong>LSAT</strong>?(Select all that apply.)Responses to this item are voluntary and will be used for statistical research purposes only.By studying the free sample questions available on LSAC’s website.By taking the free sample <strong>LSAT</strong> available on LSAC’s website.By working through official <strong>LSAT</strong> <strong>PrepTest</strong>s, ItemWise, and/or other LSAC test prep products.By using <strong>LSAT</strong> prep books or software not published by LSAC.By attending a commercial test preparation or coaching course.By attending a test preparation or coaching course offered through an undergraduate institution.Self study.Other preparation.No preparation.CERTIFYING STATEMENTPlease write the following statement. Sign and date.I certify that I am the examinee whose name appears on this answer sheet and that I amhere to take the <strong>LSAT</strong> for the sole purpose of being considered for admission to lawschool. I further certify that I will neither assist nor receive assistance from any othercandidate, and I agree not to copy, retain, or transmit examination questions in any formor discuss them with any other person.Sign your name in fullDateFOR LSAC USE ONLYSIGNATURE: TODAY’S DATE: MONTH DAY YEAR


www.cambridgelsat.comWait for the supervisor’s instructions beforeyou open the page to the topic.Please print and sign your name and write the date in the designated spaces below.Time: 35 MinutesGeneral DirectionsYou will have 35 minutes in which to plan and write an essay on the topic inside. Read the topic and the accompanying directions carefully.You will probably find it best to spend a few minutes considering the topic and organizing your thoughts before you begin writing. In your essay,be sure to develop your ideas fully, leaving time, if possible, to review what you have written. Do not write on a topic other than the onespecified. Writing on a topic of your own choice is not acceptable.No special knowledge is required or expected for this writing exercise. Law schools are interested in the reasoning, clarity, organization,language usage, and writing mechanics displayed in your essay. How well you write is more important than how much you write.Confine your essay to the blocked, lined area on the front and back of the separate Writing Sample Response Sheet. Only that area will bereproduced for law schools. Be sure that your writing is legible.Both this topic sheet and your response sheet must be turned over to the testing staffbefore you leave the room.Topic CodePrint Your Full Name HereLast First M.I.Date/ /Sign Your Name HereScratch PaperDo not write your essay in this space.


<strong>LSAT</strong> ®www.cambridgelsat.comWriting SampleTopic© <strong>2007</strong> by Law School Admission Council, Inc. All rights reserved.Directions: The scenario presented below describes two choices, either one of which can be supported on the basis of the information given.Your essay should consider both choices and argue for one over the other, based on the two specified criteria and the facts provided. Thereis no “right” or “wrong” choice: a reasonable argument can be made for either.BLZ Stores, an established men’s clothing retailer with a chain of stores in a major metropolitan area, is selecting a plan for expansion.Using the facts below, write an essay in which you argue for one of the following plans over the other based on the following two criteria:• The company wants to increase its profits.• The company wants to ensure its long-term financial stability.The “national plan” is to open a large number of men’s clothing stores throughout the country over a short period of time. In doing this,the company would incur considerable debt. It would also have to greatly increase staff and develop national marketing and distributioncapabilities. Many regional companies that adopted this strategy increased their profits dramatically. A greater number tried and failed,suffering severe financial consequences. BLZ is not well known outside its home area. Research indicates that the BLZ name is viewedpositively by those who know it. National clothing chains can offer lower prices because of their greater buying power. BLZ currently facesincreasingly heavy competition in its home region from such chains.The “regional plan” is to increase the number and size of stores in the company’s home region and upgrade their facilities, product quality,and service. This could be achieved for the most part with existing cash reserves. These upgrades would generally increase the prices that BLZcharges. In one trial store in which such changes were implemented, sales and profits have increased. The local population is growing. BLZenjoys strong customer loyalty. Regional expansion could be accomplished primarily using BLZ’s experienced and loyal staff and would allowcontinued reliance on known and trusted suppliers, contractors, and other business connections.WP-L030BScratch PaperDo not write your essay in this space.


www.cambridgelsat.comLASTNAME(Print)L0123456789012345678901234567890123456789LSAC ACCOUNT NO.0123456789012345678901234567890123456789MITESTCENTER NO.FIRSTNAME(Print)SIGNATUREDO NOT WRITEIN THIS SPACELAST 4 DIGITS OF SOCIALSECURITY/SOCIALINSURANCE NO.M M D D Y YTEST DATEWriting Sample Response Sheet01234567890123456789012345678901234567890123456789TOPIC CODE0123456789Begin your essay in the lined area below.Continue on the back if you need more space.®S C A N T R O NEliteView EM-252259-8:654321© 2013 by Law School Admission Council, Inc. All Rights Reserved.


www.cambridgelsat.com


www.cambridgelsat.comThe Practice Test37Answer Key for the Practice TestSECTION I1. A2. C3. C4. E5. E6. C7. A8. D9. E10. D11. A12. A13. D14. E15. A16. A17. D18. B19. D20. C21. D22. B23. ASECTION II1. B2. B3. D4. A5. B6. B7. D8. A9. E10. B11. C12. A13. C14. E15. D16. D17. B18. B19. A20. E21. A22. D23. C24. A25. CSECTION III1. C2. A3. E4. E5. C6. B7. B8. E9. D10. E11. E12. D13. B14. C15. C16. B17. B18. B19. A20. C21. B22. C23. D24. E25. ASECTION IV1. E2. D3. A4. E5. D6. B7. A8. A9. C10. B11. D12. C13. E14. A15. A16. A17. E18. C19. B20. E21. D22. D23. A24. B25. D26. E27. C


www.cambridgelsat.com38The Practice TestComputing Your ScoreDirections:1. Use the Answer Key on page 37 to check youranswers.2. Use the Scoring Worksheet below to compute yourraw score.3. Use the Score Conversion Chart to convert yourraw score into the 120-180 scale.Scoring Worksheet1. Enter the number of questions you answeredcorrectly in each section.SECTION I. . . . . .SECTION II . . . . .SECTION III. . . . .SECTION IV. . . . .2. Enter the sum here:NumberCorrectThis is your Raw Score.Conversion ChartFor Converting Raw Score to the 120-180 <strong>LSAT</strong>Scaled ScoreForm 8LSN75 (<strong>2007</strong>)ReportedScore180179178177176175174173172171170169168167166165164163162161160159158157156155154153152151150149148147146145144143142141140139138137136135134133132131130129128127126125124123122121120Lowest99—*9897—*9695—*9493929190898786858381807876757371696765636159575554525048464442403937353332302927262524222120191817—*160Raw ScoreHighest100—*9897—*9695—*9493929190898886858482807977757472706866646260585654535149474543413938363432312928262524232120191817—*1615*There is no raw score that will produce this scaled score for this form.

Hooray! Your file is uploaded and ready to be published.

Saved successfully!

Ooh no, something went wrong!